5
$\begingroup$

Is the following statement correct?

Let $F$ be field with characteristic not equal to $p$ which does not contain any primitive $p$th root of unity. Let $\phi_{p}(X) = f_{1}(X)f_{2}(X)\cdots f_{k}(X)$ be the factorization of $p$th cyclotomic polynomial into irreducible factors over $F$. Then $\phi_{p^2(X)} = f_{1}(X^p)f_{2}(X^p)\cdots f_{k}(X^p)$ is the factorization of $p^2$th cyclotomic polynomial into irreducible factors over $F$.

If the statement is correct, then what is the idea of the proof?

$\endgroup$
2
  • 6
    $\begingroup$ It's not true. Take $F=\mathbb{F}_2, p=1093$. Then the $f_i(X^p)$ are reducible. $\endgroup$ Oct 22, 2016 at 19:00
  • 6
    $\begingroup$ ...because 1093 is a Wieferich prime (en.wikipedia.org/wiki/Wieferich_prime). $\endgroup$
    – znt
    Oct 22, 2016 at 20:25

1 Answer 1

7
$\begingroup$

I will just elaborate on Felipe Voloch's and znt's comments.

Since $\phi_{p^2}(x) = \phi_p(x^p)$, we have the following: If $\prod f_i(x)$ is a factorization of $\phi_p(x)$ then indeed $\prod f_i(x^p)$ is a (not necessarily "final"!) factorization of $\phi_{p^2}(x)$. Hence, the question may be reformulated equivalently as follows: If $f(x)$ is the minimal polynomial of some root of $\phi_p(x)$, is $f(x^p)$ still irreducible?


(Classical) Lemma: Let $F=\mathbb{F}_q$, and let $n$ be coprime to $\text{char}(F)$. Then $\phi_{n}$ factors into $\frac{\phi(n)}{\text{ord}_n(q)}$ irreducible factor of degree $\text{ord}_n(q)$. ($\text{ord}_n(q)$ denotes the multiplicative order of $q$ modulo $n$.)

Proof: To understand the factorization of $\phi_n$ in $F$, we need to recall the following: If $\alpha \in \overline{F}$, then the degree of the minimal polynomial of $\alpha$ over $F$ (which is immediately irreducible over $F$) is the number of elements in the orbit of $\alpha$ under the action of the Frobenius $x\mapsto x^{|F|}$. Let $\alpha$ be a root of $\phi_n$ in $\overline{F}$. The orbit of $\alpha$ is of size $d$ iff $\alpha^{q^d}=\alpha$ and $\alpha^{q^{d'}} \neq \alpha$ for $d' \mid d$. In other words, if $\alpha$ is a root of unity of order dividing $q^d-1$ but not $q^{d'}-1$ for $d' \mid d$. By definition, $\alpha$ is a root of unity of order $n$. So we are looking for the minimal $d$ such that $n \mid q^d-1$. This $d$ is the (multiplicative) order of $q$ modulo $n$. Since $\phi_n$ factors into a product of some minimal polynomials, all of which must be of degree $\text{ord}_n(q)$, we are done. $\blacksquare$


It follows from the lemma that each $f_i(x)$ is of degree $\text{ord}_{p}(|F|)$, which implies that $\deg f_i(x^p) = p \cdot \text{ord}_{p}(|F|)$. The lemma also implies that the irreducible factors of $\phi_{p^2}$ are of degree $\text{ord}_{p^2}(|F|)$. So you ask: do we always have $\text{ord}_{p^2}(|F|) = p \cdot \text{ord}_{p}(|F|)$? Well, since $A \equiv 1 \bmod p \implies A^p \equiv 1 \bmod {p^2}$, it follows that $$\text{ord}_{p}(|F|) \mid \text{ord}_{p^2}(|F|) \mid p \cdot \text{ord}_{p}(|F|),$$ i.e. $$\text{ord}_{p^2}(|F|) \in \{ p \cdot \text{ord}_{p}(|F|), \text{ord}_{p}(|F|) \}.$$ Both options are possible! If, by chance, $$(*) |F|^{ \text{ord}_{p}(|F|) } \equiv 1 \bmod {p^2},$$ then $\text{ord}_{p^2}(|F|) = \text{ord}_{p}(|F|)$. This means that each $f_i(X^p)$ is reducible (factors into $p$ factors of degree $\frac{\phi(n)}{\text{ord}_{p}(|F|)}$). Since $\text{ord}_{p}(|F|) \mid p-1$ and $(p-1,p)=1$, it follows that $(*)$ is equivalent to $$|F|^{p-1} \equiv 1 \bmod {p^2}.$$ In other words, the only counterexamples to your question are primes $p$ which are "Wieferich primes in base $|F|$". It's a conjecture that for every natural number $a$, there are infinitely many Wieferich primes in base $a$. See this Wikipedia article.

$\endgroup$
2
  • $\begingroup$ Can you have any more information on this topic ? $\endgroup$ Oct 26, 2016 at 17:27
  • 1
    $\begingroup$ @Shibproshadpramanik Check out "Finite Fields" by Lidl and Niederreiter. Chapter 2 contains an excellent section on cyclotomic polynomials and roots of unity in finite fields. $\endgroup$ Oct 28, 2016 at 8:39

Your Answer

By clicking “Post Your Answer”, you agree to our terms of service and acknowledge you have read our privacy policy.

Not the answer you're looking for? Browse other questions tagged or ask your own question.